Fourier transform, complex exponential and infinity

Click For Summary
The discussion focuses on evaluating the Fourier transform of a signal, specifically addressing the integral's bounds from 0 to infinity due to the signal being zero for negative time. The user encounters difficulty in evaluating the limit as t approaches infinity for a complex expression. Another participant clarifies that the expression can be rewritten using Euler's formula, separating it into real and imaginary components. They conclude that the limit at infinity results in zero due to the exponential decay of the real part. The exchange emphasizes the importance of understanding complex exponentials in the context of Fourier transforms.
fishingspree2
Messages
138
Reaction score
0
I'm taking the Fourier transform of a signal. This integral has bounds from -∞ to ∞, but since the signal is 0 for negative t, the bounds become 0 to ∞

doing the integration, the antiderivative I get is et*(-3-jω+2j) where j is sqrt(-1)

Now I have to evaluate this at t=infinity (since it is a proper integral)...I don't really know how to do this since (-3-jω+2j) is a complex number.

infinity times a negative number is negative infinity
infinity times a positive number is positive infinity
infinity times a complex number?

any help will be appreciated,

thank you very much
 
Last edited:
Physics news on Phys.org
Looks like you have:

e^{-3t+it(2-w)}\biggr|_0^{\infty}

where I'm using i cus' I'm not an engineer but same dif. Now assume w is a real number, then we could write:

e^{-3t}e^{it(2-w)}=e^{-3t}\left(\cos(t(2-w))+i\sin(t(2-w)\right)\biggr|_0^{\infty}

so that because of the e^{-3t} real part, at the upper limit, that is zero and at the lower limit it's one.
 
thank you! got it!
 
Question: A clock's minute hand has length 4 and its hour hand has length 3. What is the distance between the tips at the moment when it is increasing most rapidly?(Putnam Exam Question) Answer: Making assumption that both the hands moves at constant angular velocities, the answer is ## \sqrt{7} .## But don't you think this assumption is somewhat doubtful and wrong?

Similar threads

  • · Replies 5 ·
Replies
5
Views
2K
Replies
2
Views
2K
  • · Replies 6 ·
Replies
6
Views
3K
  • · Replies 6 ·
Replies
6
Views
3K
Replies
8
Views
2K
  • · Replies 3 ·
Replies
3
Views
2K
  • · Replies 3 ·
Replies
3
Views
3K
  • · Replies 3 ·
Replies
3
Views
2K
  • · Replies 9 ·
Replies
9
Views
2K
  • · Replies 9 ·
Replies
9
Views
2K